Verbal problems from the *free* official practice tests and
problems from mba.com
sheetal
 
 

The higher the level of certain vitamins and minerals

by sheetal Sat Mar 01, 2008 7:57 pm

Source: GMATPrep

The higher the level of certain vitamins and minerals in the bloodstream, the better a person's lung function, as measured by the amount of air the person can expel in one second. The lung function of smokers is significantly worse, on average, than that of nonsmoker. Clearly, therefore, one way for smokers to improve their lung function is for them to increase their intake of foods that are rich in these helpful vitamins and minerals.

Which of the following is an assumption on which this argument depends?

A) Smokers are less likely than nonsmokers to have diets that are rich in vitamins and minerals

B) The lung function of smokers whose diet are rich in those vitamins and minerals is generally better than that of nonsmokers with comparable diets

C) People whose diets are deficient in those vitamins and minerals do not typically have other health problems in addition to diminished lung function.

D) Stopping smoking will not typically improve lung function more than any diet changes can.

E) Smoking does not introduce into the body chemicals that prevent the helpful vitamins and minerals from entering the bloodstream

I could identify A, D and E to be contenders. Finally, I went with D. Can the instructors explain why is E the best answer. What is wrong with A and D ?

Thanks,
Sheetal
RonPurewal
Students
 
Posts: 19744
Joined: Tue Aug 14, 2007 8:23 am
 

by RonPurewal Wed Mar 05, 2008 6:16 am

the first premise states, as fact, the following correlation:
higher vita/min in bloodstream <--> better lung function

that correlation is presented as an established fact, and so we are not allowed to question it. however, notice that the correlation is only asserted for vitamins and minerals in the bloodstream - i.e., they have to get into the bloodstream in the first place, not merely be ingested.

--

notice also that the correlation is a FACT, regardless of whether the person in question is a smoker. in other words, if smokers get higher levels of the vitamins/minerals into their bloodstreams, then a fortiori they will also have better lung function.

the argument is therefore airtight, provided the smokers can get the vitamins and minerals into their bloodstreams to begin with. that's the only missing link here - all that's mentioned in the argument is intake, which is an entirely separate matter from absorption into the bloodstream.

choice e is correct because it takes care of the connection between intake and absorption, by establishing that the smokers' intake of vitamins and minerals will actually make it into the bloodstream.

--

choice a is irrelevant; it speaks of vitamins and minerals in general, not the special vitamins and minerals mentioned in the passage. what's more, if it does anything it actually strengthens the conclusion (because adding vitamins/minerals to a vitamin- and mineral-poor diet will likely have greater consequences than will adding them to a diet that is already vitamin- and mineral-rich)!

--

choice d is irrelevant, as cessation of smoking doesn't affect the issue (the effect of the vitamins/minerals) at all.
Guest
 
 

by Guest Wed Mar 05, 2008 9:41 am

Thanks Ron for the great explanation.
StaceyKoprince
ManhattanGMAT Staff
 
Posts: 9349
Joined: Wed Oct 19, 2005 9:05 am
Location: Montreal
 

by StaceyKoprince Wed Mar 12, 2008 3:55 pm

On behalf of Ron, you're welcome!
Stacey Koprince
Instructor
Director, Content & Curriculum
ManhattanPrep
priyankadhawan223
Forum Guests
 
Posts: 6
Joined: Sat Nov 10, 2012 1:23 pm
 

Re: The higher the level of certain vitamins and minerals

by priyankadhawan223 Wed Nov 21, 2012 12:11 am

hi
i juust wanted to ask, how hard is this question?
tim
Course Students
 
Posts: 5665
Joined: Tue Sep 11, 2007 9:08 am
Location: Southwest Airlines, seat 21C
 

Re: The higher the level of certain vitamins and minerals

by tim Sat Nov 24, 2012 2:55 pm

i've found that the answers to this type of question are almost never useful in preparing someone for the GMAT, so i'll decline to provide a guess as to difficulty level unless you can tell us how you will use that information to better prepare yourself for the GMAT.. :)
Tim Sanders
Manhattan GMAT Instructor

Follow this link for some important tips to get the most out of your forum experience:
https://www.manhattanprep.com/gmat/forums/a-few-tips-t31405.html
AbhilashM94
Students
 
Posts: 53
Joined: Sun Apr 27, 2014 1:26 am
 

Re: The higher the level of certain vitamins and minerals

by AbhilashM94 Sun Jul 06, 2014 6:59 am

1) Can someone explain why B is wrong?
Ron: Need some help buddy

2) Also is this really a GMAT PREP question?

3) Is the best way to tackle assumption question by inverting it and see which as the best impact? By that logic maybe E is better that B.
RonPurewal
Students
 
Posts: 19744
Joined: Tue Aug 14, 2007 8:23 am
 

Re: The higher the level of certain vitamins and minerals

by RonPurewal Wed Jul 09, 2014 12:31 pm

AbhilashM94 Wrote:1) Can someone explain why B is wrong?
Ron: Need some help buddy


"Buddy"? Hmm.

The best way to approach this issue is for you to say why you think B might be right!

B says that those smokers have better lung function than nonsmokers who are on exactly the same diet.
!!!
Not only is this unnecessary, but, on consideration, it's so farfetched as to be pretty much complete nonsense.
RonPurewal
Students
 
Posts: 19744
Joined: Tue Aug 14, 2007 8:23 am
 

Re: The higher the level of certain vitamins and minerals

by RonPurewal Wed Jul 09, 2014 12:31 pm

3) Is the best way to tackle assumption question by inverting it and see which as the best impact? By that logic maybe E is better that B.


There's no such thing as "the best way".
If method #1 and method #2 both solve a problem, then each is just as good as the other.
gmatkiller_24
Students
 
Posts: 103
Joined: Sun Jun 10, 2012 8:33 pm
 

Re: The higher the level of certain vitamins and minerals

by gmatkiller_24 Sun Mar 08, 2015 4:24 pm

RonPurewal Wrote:
AbhilashM94 Wrote:1) Can someone explain why B is wrong?
Ron: Need some help buddy


"Buddy"? Hmm.

The best way to approach this issue is for you to say why you think B might be right!

B says that those smokers have better lung function than nonsmokers who are on exactly the same diet.
!!!
Not only is this unnecessary, but, on consideration, it's so farfetched as to be pretty much complete nonsense.


it seems to be choice B has somewhat weaken the whole argument by demonstrating that other factors ( instead of vitamin stuff) are contributing to smokers' better lung function?

is it right? Thanks!
RonPurewal
Students
 
Posts: 19744
Joined: Tue Aug 14, 2007 8:23 am
 

Re: The higher the level of certain vitamins and minerals

by RonPurewal Tue Mar 10, 2015 3:28 am

this is an assumption question, so "weakens the argument" is not a pertinent concept.
sh.pradeep
Students
 
Posts: 7
Joined: Sat Dec 17, 2011 4:57 pm
 

Re:

by sh.pradeep Thu May 07, 2015 9:15 pm

RonPurewal Wrote:the first premise states, as fact, the following correlation:
higher vita/min in bloodstream <--> better lung function

that correlation is presented as an established fact, and so we are not allowed to question it. however, notice that the correlation is only asserted for vitamins and minerals in the bloodstream - i.e., they have to get into the bloodstream in the first place, not merely be ingested.

--

notice also that the correlation is a FACT, regardless of whether the person in question is a smoker. in other words, if smokers get higher levels of the vitamins/minerals into their bloodstreams, then a fortiori they will also have better lung function.

the argument is therefore airtight, provided the smokers can get the vitamins and minerals into their bloodstreams to begin with. that's the only missing link here - all that's mentioned in the argument is intake, which is an entirely separate matter from absorption into the bloodstream.

choice e is correct because it takes care of the connection between intake and absorption, by establishing that the smokers' intake of vitamins and minerals will actually make it into the bloodstream.

--

choice a is irrelevant; it speaks of vitamins and minerals in general, not the special vitamins and minerals mentioned in the passage. what's more, if it does anything it actually strengthens the conclusion (because adding vitamins/minerals to a vitamin- and mineral-poor diet will likely have greater consequences than will adding them to a diet that is already vitamin- and mineral-rich)!

--

choice d is irrelevant, as cessation of smoking doesn't affect the issue (the effect of the vitamins/minerals) at all.



Hi Ron/Experts,

Thanks a lot for the excellent reply, but I still can't see why D is wrong.
My line of thinking is : If someone stops smoking, then the harmful chemicals, if any, ( as cited in E) will also stop and hence higher vitamins and minerals in the bloodstream. In that way stopping smoking is doing good for lung function.

Some thoughts please

Regards
RonPurewal
Students
 
Posts: 19744
Joined: Tue Aug 14, 2007 8:23 am
 

Re: Re:

by RonPurewal Fri May 08, 2015 10:09 am

there are at least three very major problems here.

1/
you seem to be unclear on what "assumption" means.
an "assumption" is something that's absolutely necessary for the argument to work. (perhaps more intuitively, if an assumption is false, then the entire argument is destroyed.)
your explanation simply supposes a number of things that there's no reason to presume. (in other words, making stuff up out of nowhere).

--

2/
you can't use the content of other answer choices when you consider a choice!
in your discussion of choice D, you're referring to choice E.
nope.

--

3/
you should go back and read the words more carefully, because choice D actually says
Stopping smoking will not typically improve lung function more than xxxxx

you've somehow twisted this into a statement that stopping smoking WILL improve lung function. that's not what the words say. (in fact, that's much closer to the opposite of what the words say.)
sh.pradeep
Students
 
Posts: 7
Joined: Sat Dec 17, 2011 4:57 pm
 

Re: Re:

by sh.pradeep Fri May 08, 2015 7:47 pm

RonPurewal Wrote:there are at least three very major problems here.

1/
you seem to be unclear on what "assumption" means.
an "assumption" is something that's absolutely necessary for the argument to work. (perhaps more intuitively, if an assumption is false, then the entire argument is destroyed.)
your explanation simply supposes a number of things that there's no reason to presume. (in other words, making stuff up out of nowhere).

--

2/
you can't use the content of other answer choices when you consider a choice!
in your discussion of choice D, you're referring to choice E.
nope.

--

3/
you should go back and read the words more carefully, because choice D actually says
Stopping smoking will not typically improve lung function more than xxxxx

you've somehow twisted this into a statement that stopping smoking WILL improve lung function. that's not what the words say. (in fact, that's much closer to the opposite of what the words say.)



I am so grateful for your early reply.
RonPurewal
Students
 
Posts: 19744
Joined: Tue Aug 14, 2007 8:23 am
 

Re: Re:

by RonPurewal Wed May 13, 2015 5:19 am

no problem.